LSAT and Law School Admissions Forum

Get expert LSAT preparation and law school admissions advice from PowerScore Test Preparation.

 Administrator
PowerScore Staff
  • PowerScore Staff
  • Posts: 8917
  • Joined: Feb 02, 2011
|
#26256
Complete Question Explanation
(The complete setup for this game can be found here: lsat/viewtopic.php?t=10917)

The correct answer choice is (A)

Because this question requires that we place a variable in a specific position (Z on Thursday), it would be best to create a Linear setup, rather than a Sequencing chain:
June15_game_3_#15_diagram_1.png
If Z is on Thursday, we need to ensure than both F and K are scheduled earlier than Thursday (third and fourth rules). Furthermore, since only L, G, and Z can be on duty on Saturday, it follows that L and G must share that duty:
June15_game_3_#15_diagram_2.png
Note that the last rule has no effect on this solution, because scheduling L on Saturday is a necessary condition for F to be scheduled earlier than L. The necessary condition is clearly satisfied here, warranting no further review.

Answer choice (A): This is the correct answer choice, because F is on duty on either Monday or Tuesday, whereas L is on duty on Saturday.

Answer choice (B) is incorrect, because the relationship between F and H is still unknown. It is possible that F is on duty on Monday.

Answer choice (C) is incorrect, because the relationship between H and M is unknown.

Answer choice (D) is incorrect, because H could be on duty on Friday.

Answer choice (E) is incorrect, because the relationship between K and M is unknown.
You do not have the required permissions to view the files attached to this post.

Get the most out of your LSAT Prep Plus subscription.

Analyze and track your performance with our Testing and Analytics Package.